[別館]球面倶楽部零八式markIISR

東大入試数学中心。解説なので解答としては不十分。出題年度で並ぶようにしている。大人の解法やうまい解法は極めて主観的に決めている。

1988年(昭和63年)東京工業大学-数学[5]

2023.07.26記

[5] \displaystyle\lim_{n\to\infty}\left(\dfrac{{}_{3n}\mbox{C}_{n}}{{}_{2n}\mbox{C}_{n}}\right)^{\frac{1}{n}} を求めよ.

2023.07.26記

本問のテーマ
Stirling の公式(Stirling の近似)
シャノンエントロピー

n,m が大きいとき,
スターリングの公式 \log N!\sim N\log N-N から
\log {}_n\mbox{C}_m\sim n\log n - n - m\log m + m - (n-m)\log (n-m) +n-m
=-m\log\dfrac{m}{n}-(n-m)\log\dfrac{n-m}{n}=n H\left(\dfrac{m}{n}\right)
となる.ここで H(x) は2元アルファベットのシャノンエントロピーである.

なお,計算上は
\log {}_n\mbox{C}_m=n\log n -  m\log m - (n-m)\log (n-m)
つまり
{}_n\mbox{C}_m\dfrac{n^n}{m^m (n-m)^{n-m}}
(これは比の値が1に近づかないので「\sim」にはならないが、大抵 \dfrac{1}{n} 乗根と組合せるので問題にならないことが多い,後述)の方が速いことが多い.

[大人の解答]
a_n=\left(\dfrac{{}_{3n}\mbox{C}_{n}}{{}_{2n}\mbox{C}_{n}}\right)^{\frac{1}{n}} とおくと
\log a_n=\dfrac{1}{n}\left(3n H\left(\dfrac{1}{3}\right)-2n H\left(\dfrac{1}{2}\right)\right)
=3H\left(\dfrac{1}{3}\right)-2H\left(\dfrac{1}{2}\right)
=3\left(\dfrac{1}{3}\log 3+\dfrac{2}{3}\log\dfrac{3}{2}\right)-2\left(\dfrac{1}{2}\log 2+\dfrac{1}{2}\log 2\right)
=3\log 3-4\log 2=\log\dfrac{27}{16}
となり,求める極限は \dfrac{27}{16} となる.

見易いように H(x) を経由したが,経由せずに
\displaystyle\left(\dfrac{{}_{3n}\mbox{C}_{n}}{{}_{2n}\mbox{C}_{n}}\right)^{\frac{1}{n}}\mbox{≒}\left(\dfrac{\dfrac{(3n)^{3n}}{n^{n}\cdot (2n)^{2n}}}{\dfrac{(2n)^{2n}}{n^{n}\cdot n^{n}}}\right)^{\frac{1}{n}}=\dfrac{\dfrac{(3)^{3}}{1^{1}\cdot (2)^{2}}}{\dfrac{(2)^{2}}{1^{1}\cdot 1^{1}}}=\dfrac{3^3}{2^4}=\dfrac{27}{16}
とした方が速い.

なお,スターリングの公式 \log N!\sim N\log N-N は簡易版とも呼ばれる.
というのも,この近似では \log N!\sim N\log N-N であるものの
 N!\not\sim \left(\dfrac{N}{e}\right)^n
である.より精密には
 N!\sim \sqrt{2\pi N}\left(\dfrac{N}{e}\right)^N
となる.

なお,

2015年(平成27年)大阪大学-前期専門数学[2]

 N!\sim \sqrt{2\pi N}\left(\dfrac{N}{e}\right)^N に関する出題である.これを用いると
{}_n\mbox{C}_m\sim\dfrac{1}{\sqrt{2\pi}}\cdot \dfrac{n^{n+\frac{1}{2}}}{m^{m+\frac{1}{2}}(n-m)^{n-m+\frac{1}{2}}}
となることがわかる.

精密化における違い \sqrt{\dfrac{n}{2\pi m(n-m)}}は,本問では
\dfrac{\sqrt{\dfrac{3n}{2\pi\cdot 2n^2}}}{\sqrt{\dfrac{2n}{2\pi n^2}}}=\sqrt{\dfrac{3}{4}}
となるが,これは\dfrac{1}{n} 乗されるので1に近づく故,本問では問題にならない差異となる.

[解答]
a_n=\left(\dfrac{{}_{3n}\mbox{C}_{n}}{{}_{2n}\mbox{C}_{n}}\right)^{\frac{1}{n}} とおくと
\log a_n =\dfrac{1}{n}\log\dfrac{{}_{3n}\mbox{P}_n}{{}_{2n}\mbox{P}_n}=\displaystyle\dfrac{1}{n}\sum_{k=1}^n\log\dfrac{2+\dfrac{k}{n}}{1+\dfrac{k}{n}}
により,
\displaystyle\lim_{n\to\infty}\log a_n =\int_0^1\log\dfrac{2+x}{1+x} dx=\Bigl[ (x+2)\log(x+2)-(x+1)\log(x+1)\Bigr]_0^1=3\log 3-4\log 2=\log\dfrac{27}{16}
となり,求める極限は \log\dfrac{27}{16} となる.

1968年(昭和43年)東京工業大学-数学[5]

も参照のこと.